Solving Coefficient not using Fourier Series coefficient

Click For Summary
The discussion centers on solving Laplace's equation with specific boundary conditions, leading to a general solution expressed as a series involving sine and hyperbolic sine functions. The user derived the equation ∑An sin(π n y )sinh (π n) = y(1-y) and seeks to determine the coefficients An without using Fourier series methods. A response suggests that finding An without Fourier coefficients is unlikely, as the method typically relies on identifying specific terms in the series. The example provided illustrates that in certain cases, coefficients can be directly determined from the equation. Ultimately, the consensus indicates that the task may not be feasible without Fourier series.
Alana02011114
Messages
1
Reaction score
0
Given the Laplace's equation with several boundary conditions. finally i got the general solution u(x,t).
One of the condition is that:
u(1,y)=y(1-y)

After working on this I finally got:
∑An sin(π n y )sinh (π n) = y(1-y)

However, i was asked to find An, by not using Fourier series coefficient, Is it possible to do so? Cheers
 
Physics news on Phys.org
Alana02011114 said:
Given the Laplace's equation with several boundary conditions. finally i got the general solution u(x,t).
One of the condition is that:
u(1,y)=y(1-y)

After working on this I finally got:
∑An sin(π n y )sinh (π n) = y(1-y)

However, i was asked to find An, by not using Fourier series coefficient, Is it possible to do so? Cheers

No, I don't think so. That hint usually arises in a situation where, if your equation were$$\sum_{n=1}^\infty A_n\sinh(\pi n)\sin(n\pi y) = 5\sin(3\pi y)$$where you could immediately say$$A_3 \sinh(3\pi) = 5$$ and all the other ##A_n=0##.
 
Question: A clock's minute hand has length 4 and its hour hand has length 3. What is the distance between the tips at the moment when it is increasing most rapidly?(Putnam Exam Question) Answer: Making assumption that both the hands moves at constant angular velocities, the answer is ## \sqrt{7} .## But don't you think this assumption is somewhat doubtful and wrong?

Similar threads

  • · Replies 3 ·
Replies
3
Views
2K
Replies
3
Views
1K
  • · Replies 2 ·
Replies
2
Views
1K
  • · Replies 2 ·
Replies
2
Views
2K
  • · Replies 6 ·
Replies
6
Views
2K
  • · Replies 6 ·
Replies
6
Views
1K
  • · Replies 16 ·
Replies
16
Views
2K
  • · Replies 1 ·
Replies
1
Views
1K
Replies
3
Views
2K
  • · Replies 1 ·
Replies
1
Views
2K